LSAT and Law School Admissions Forum

Get expert LSAT preparation and law school admissions advice from PowerScore Test Preparation.

User avatar
 Dave Killoran
PowerScore Staff
  • PowerScore Staff
  • Posts: 5852
  • Joined: Mar 25, 2011
|
#40822
Complete Question Explanation
(The complete setup for this game can be found here: lsat/viewtopic.php?t=15545)

The correct answer choice is (A)

If I and P are both assigned to the middle floor, then Template #1 is in effect, with T added to the middle floor from the first rule:
PT59_Game_#1_#2_diagram 1.png
This diagram eliminates answer choices (B), (C), (D), and (E):

Answer choice (B): H must be assigned to the top floor per the second rule.

Answer choice (C): L must be assigned to the bottom floor.

Answer choice (D): Because L is assigned to the bottom floor, from the third rule no other department can be assigned to the bottom floor.

Answer choice (E): From the first rule T must be assigned to the middle floor.

Thus, answer choice (A) is correct. Also, remember that Could be True questions often address unfixed variables and this would allow you to focus on answer choices (A) and (D).
You do not have the required permissions to view the files attached to this post.

Get the most out of your LSAT Prep Plus subscription.

Analyze and track your performance with our Testing and Analytics Package.